PT5.S3.10 - The reforms to improve the quality of public education

Could someone explain why D is wrong and A is right?

I initially chose D because it said "Parents are given vouchers" and "to attract students."

I thought students could be attracted, but then that doesn't mean that they would attend the school, because it's up to the parents.

Comments

  • Ashley2018-1Ashley2018-1 Alum Member
    2249 karma

    Well, the issue isn't whether or not the children will attend the school. The argument is assuming the parents will use their vouchers to select schools based upon how good their academic offerings are since the argument uses this as evidence to conclude the schools will be FORCED to improve their academics. If it is true that the parents will not choose their schools based on quality of academics and instead on other factors such as location and sports offering (which is perfectly reasonable), then the conclusion is not necessarily true.

  • Auntie2020Auntie2020 Member
    552 karma

    Really great explanation. Thank you @"ashley.tien"

Sign In or Register to comment.